Difference between revisions of "2021 Fall AMC 10A Problems/Problem 14"

(Solution)
Line 8: Line 8:
  
  
==Solution==
+
==Solution (Graphing)==
 
 
 
 
 
 
 
 
 
 
  
 
==See Also==
 
==See Also==
 
{{AMC10 box|year=2021 Fall|ab=A|num-b=13|num-a=15}}
 
{{AMC10 box|year=2021 Fall|ab=A|num-b=13|num-a=15}}
 
{{MAA Notice}}
 
{{MAA Notice}}

Revision as of 20:27, 23 November 2021

Problem

How many ordered pairs $(x,y)$ of real numbers satisfy the following system of equations? \[x^2+3y=9\] \[(|x|+|y|-4)^2 = 1\]

$\textbf{(A )} 1 \qquad\textbf{(B) } 2 \qquad\textbf{(C) } 3 \qquad\textbf{(D) } 5 \qquad\textbf{(E) } 7$


Solution (Graphing)

See Also

2021 Fall AMC 10A (ProblemsAnswer KeyResources)
Preceded by
Problem 13
Followed by
Problem 15
1 2 3 4 5 6 7 8 9 10 11 12 13 14 15 16 17 18 19 20 21 22 23 24 25
All AMC 10 Problems and Solutions

The problems on this page are copyrighted by the Mathematical Association of America's American Mathematics Competitions. AMC logo.png